Find the volume of the figure below. Round to the nearest tenth.

Find The Volume Of The Figure Below. Round To The Nearest Tenth.

Answers

Answer 1

Answer:

The volume of the figure given is:

3057,57 cm^3

Step-by-step explanation:

To find the volume of the figure in the picture, first, you must divide the figure in the two known figures there: a cylinder and a rectangular prism, now you can find the volume of each figure separated and at the last add the two volumes.

Volume of the cylinder.

The formula to find the volume of a cylinder is:

Volume of a cylinder = PI * r^2 * h

Where:

PI = 3,1416 approximately (In this case I´ll use the number without approximation, you can find this in the calculator).

r = radius (the distance from a side of the circle to the center of this, in this case, is 7 cm)

h = height (in this case is 7 cm too, you can find these values in the figure given)

Now we must replace the values:

Volume of a cylinder = PI * r^2 * h Volume of a cylinder = PI * (7 cm)^2 * 7 cm Volume of a cylinder = 1077,57 cm^3 approximately.

Volume of the rectangular prism.

The volume of a rectangular prism is actually easy, you only must multiply the width, the length, and the height, so, the formula is:

Volume of a rectangular prism = width * length * height.

When we replace the values given in the figure:

Volume of a rectangular prism = 11 cm * 20 cm * 9 cm  Volume of a rectangular prism = 1980 cm^3

At the last, we add the two volumes found:

Volume of the figure = 1077,57 cm^3 + 1980 cm^3 Volume of figure = 3057,57 cm^3

Related Questions

8x + 9y = 9 3x – 5y = 20 a. (3.36, –1.99) c. (0, –1.99) b. (–1.11, 2.79) d. (3.36, 2.79)

Answers

Answer:

x = 225/67 , y = -133/67 or decimal x≈3.3582 and y≈-1.9851, thus a. is the answer rounded.

Step-by-step explanation:

Solve the following system:

{8 x + 9 y = 9 | (equation 1)

3 x - 5 y = 20 | (equation 2)

Subtract 3/8 × (equation 1) from equation 2:

{8 x + 9 y = 9 | (equation 1)

0 x - (67 y)/8 = 133/8 | (equation 2)

Multiply equation 2 by 8:

{8 x + 9 y = 9 | (equation 1)

0 x - 67 y = 133 | (equation 2)

Divide equation 2 by -67:

{8 x + 9 y = 9 | (equation 1)

0 x+y = -133/67 | (equation 2)

Subtract 9 × (equation 2) from equation 1:

{8 x+0 y = 1800/67 | (equation 1)

0 x+y = -133/67 | (equation 2)

Divide equation 1 by 8:

{x+0 y = 225/67 | (equation 1)

0 x+y = -133/67 | (equation 2)

Collect results:

Answer: {x = 225/67 , y = -133/67

Can someone help me out, I'm having trouble on these two questions.

Answers

Answer:

Step-by-step explanation:

7,A=9*6sin30°

=54*0.5

A= 27

8,A=6*10sin60°

A= 60*0.86

A=52....... approximately

9, A=1/2bh

A=0.5*x*xroot 3

A=root3/2 x^2

You can find x using trigonometric values


GUYS PLS I NEED HELP

Answers

Answer:

z = 75°

Step-by-step explanation:

You can see that CGE and FGD are vertical angles.

FGD = 15°

Now you have a 90° angle, a 15° angle, and a z° angle. They add up to 180°

105° + z° = 180°

z = 75°

Answer:

75 degrees...

Step-by-step explanation:

if you look at the diagram, the angle with 15 degrees is the same as the one opposite!

So we know that it is on a right angled corner, so all the angles here should add up to 90 degrees!

Now we do 90 degrees - 15 degrees

This gives us 75 degrees!

So x = 75 degrees!

HOPE THIS HELPS!!

Expand the binomial using Pascal's triangle
(3x+5)

Answers

Answer:

Step-by-step explanation:

0.           1

1.          1  1

2.         1  2  1

3.        1  3  3  1

4.       1  4  6  4  1

5.      1  5  10  10  5  1

6.     1  6  15  20  15  6  1

7.    1  7  21  35  35  21  7  1

8.   1  8  28  56  70  56  28  8  1

Please help!!! WILL MARK BRAINLIEST ANSWER!!! A medical researcher designs a study that will investigate the effects of a
new drug on lowering patient's cholesterol level. The researcher randomly
selects participants for two groups. One group is given the new drug and the other group is given a placebo. Any person, within driving distance of the hospital, is allowed to participate and will be given a monetary compensation. Each sample consists of 5 patients. Select all weaknesses in the design.

Answers

 b. more groups should have been

Answer:

C AND D

Step-by-step explanation:

list the first 3 terms, then evaluate

Answers

Answer:

[tex]\sum\limits_{i=1}^{10}(1-2i)=-100[/tex].

Step-by-step explanation:

The given expression is

[tex]\sum\limits_{i=1}^{10}(1-2i)[/tex]

We need to find first 3 terms, then we have evaluate the value.

Here,  

[tex]a_i=1-2i[/tex]

For i=1,

[tex]a_1=1-2(1)=1-2=-1[/tex]

For i=2,

[tex]a_2=1-2(2)=1-4=-3[/tex]

For i=3,

[tex]a_3=1-2(3)=1-6=-5[/tex]

So first three terms are -1,-3,-5. It is an AP with first term -1 and common difference -2. So, next 7 terms are -7,-9,-11,-13,-15,-17,-19.

Now,  

[tex]\sum\limits_{i=1}^{10}(1-2i)=(-1)+(-3)+(-5)+(-7)+(-9)+(-11)+(-13)+(-15)+(-17)+(-19)[/tex]

[tex]\sum\limits_{i=1}^{10}(1-2i)=-100[/tex]

Therefore, the value of [tex]\sum\limits_{i=1}^{10}(1-2i)[/tex] is -100.

please need help I will be MARKING as BRIANILIST, thank you so much. ​

Answers

Answer:

Here is the mpg for each 6 cylinder:

(23), (24), (30), (23) ,(21), (20) ,(23), (21) ,(23), (22)

i : between 20-24 = 9/10

ii between 25-29 = 0/10

iii at least 30 mpg = 1/10

c- 38, 28,25,26,24,25

d- the mileage of six cylinder car is less than the mileage of four cylinder cars using the same amount of fuel. it is somehow accurate, other factor that affects is the model of the car and how big it is.

Find m∠YOZ in the figure if m∠XOZ = 86°.

Question 1 options:

A)
65°

B)


C)
42°

D)
43°

Answers

One part of the angle is 3x, and the other part is 6x+23, so putting them together gets 9x+23=86. Subtract the 23 and we get 9x=63. We know that 9•7=63, so x=7, and now we can solve the angle. 6x=42, and 42+23=65, so the answer is:

A) 65°

Answer:

A. 65

Step-by-step explanation:

First, you would need to find x.

You would combine both equations it would become, 6x+23+3x=86.

Then, combine like terms, 9x+23=86.

And subtract 23 from both sides, 9x = 63.

Lastly, divide both sides by 9.

x = 7

Now with the equation 6x+23, you replace x with 7, 6(7)+23.

Then multiply 42+23, and add 42+23= 65.


If the graph of the polynomial intersects the x-axis at two points, then the
polynomial has how many
zeros.

Answers

Answer:

Two zeroes

Step-by-step explanation:

A zero is where the graph meets the x-axis. If it hits the axis twice, there are two zeroes.

Given that: 2√2−√5 / 3√2+2√5 = =

Answers

Answer:

7√10/2 - 11

Step-by-step explanation:

(2√2−√5) / (3√2+2√5)=

(3√2-2√5)(2√2−√5)/(3√2+2√5)(3√2+-2√5)=

(12+10-3√10-4√10)/(18-20)=

(22-7√10)/-2= 7√10/2 - 11

9) Find the midpoint of the line segment with the given endpoints: (6,3), (-2,7)
A. (-10, 11)
B. (2,5)
C. (4,-2)
D. (-1,0)

Answers

Answer:

B. (2,5)

Step-by-step explanation:

Given

Points:  (6,3), (-2,7)

Required

Midpoint

Midpoint is calculated as follows:

[tex](x,y) = (\frac{x_1+x_2}{2},\frac{y_1+y_2}{2})[/tex]

Where

[tex](x_1,y_1) = (6,3)[/tex]

[tex](x_2,y_2) = (-2,7)[/tex]

So,

[tex]x_1 = 6; y_1 = 3; x_2 = -2; y_2 = 7[/tex]

By substituting these values in the given formula

[tex](x,y) = (\frac{6-2}{2},\frac{3+7}{2})[/tex]

[tex](x,y) = (\frac{4}{2},\frac{10}{2})[/tex]

[tex](x,y) = (2,5)[/tex]

Hence, the midpoint of the line segment is (2,5)

Could you plz help me with this one question

Answers

The answer is 90

Hope this helped

Answer:

90 degrees.

Step-by-step explanation:

Add all of the degrees together 270 plus 90 its makes a 360 degree.

pls help show ALL work

Answers

Answer:

B. 5.81

Step-by-step explanation:

So to get this answer, you'd first find how much square inches are on the cheese. We know its 9/10 of the pizza and the pizza is 215 square inches. To find out what is 9/10 of the pizza you'd multiply 215 by 9 then divide by 10. That'd leave you with 193.5 then you multiply that by the cost $0.03 then get 5.805 which rounds up to 5.81.

How many water do you need to mix with 200ml of shampoo? How many water should you mix with 500ml of shampoo? If the correct amount of water is mixed with 400ml of shampoo will the mixture fit in a 2500ml container?

Answers

Answer:

200 mL shampoo - 1050 ml water

500 mL shampoo - 2625 mL water

Step-by-step explanation:

1) 400 ml shampoo , should be mixed 2500 - 400 = 2100 mL of water.

2) 400 mL shampoo - 2100 mL water

    200 mL shampoo -  x mL water

x = 200*2100/400 = 1050 mL water

3) 400 mL shampoo - 2100 mL water

   500 mL shampoo - x  mL water

  x = 500*2100/400 = 2625 mL water

The table represents a liner function. What is the slope of the function?
O-3
O-2
O 3
O 4

Answers

Answer:

The slope is 3.

Step-by-step explanation:

Let's use the slope formula to calculate the slope of this function. Remember, slope equals rise over run, or the difference between y coordinates divided by the difference between x coordinates of 2 points on the graph.

Let's use the last 2 points in the table: (1, 7) and (2, 10)

[tex]\frac{y_{2} - y_{1} }{x_{2} - x_{1} }[/tex]

= [tex]\frac{10 - 7}{2 - 1}[/tex]

= [tex]\frac{3}{1}[/tex]

The slope of this function is 3!

Hope this helps :) Feel free to ask me any questions!

does anyone know the answer to this ?

Answers

Answer:

the horse walked 20 feet away i think

Step-by-step explanation:

i say this because it shows an arrow from a horse to the 20 square feet

Answer:

Step-by-step explanation:

it is a three quarter circle with radius=7.1 ft

reqd. area=3/4 πr²

=3/4×π×7.1²

≈3/4×3.14×7.1²

≈118.72 ft²

Which expression represents the greatest common factor (GCF) of 105 and 126? A. 3 x 7 B. 2 x 5 x 7 C. 2 x 3 x 7 D. 5 x 7
(HELP AS FAST AS YOU CAN PLEASE!!!!!)

Answers

Answer:

Step-by-step explanation:

factors of 105:

1, 3, 5, 7, 15, 21, 35, 105

factors of 126:

1, 2, 3, 6, 7, 9, 14, 18, 21, 42, 63, 126

GCF: 3*7 or 21

The  GCF ( 105, 126) =  3 *7 =21.

What is GCF?

The greatest common factor (GCF) of a set of numbers is the largest factor that all the numbers share.

Given:

105 = 3 * 5 * 7

126 = 2 * 3* 3 *7

So, GCF ( 105, 126) =  3 *7 =21

Learn more about GCF here:

https://brainly.com/question/9737217

#SPJ2

The number 16 has two square roots: 4 and _

Answers

Answer:

4

Step-by-step explanation:

4 times 4 = 16

Answer:

sqrt(16) = +4 and -4

Step-by-step explanation:

A square root has a positive and a negative square root

sqrt(16) = +4 and -4

1
0 5 5 8
2
2 5 8
3
5 6
1 | O means 10
Match the stem-and-leaf plot to the correct set of data.
A. 10, 15, 15, 18, 22, 25, 28, 35, 36
B. 0,1,3,5,6,8
C. 1.0, 1.5, 1.5, 1.8, 2.2, 2.5, 2.8, 3.5, 3.6
D. 1.0558, 2.258, 3.56​

Answers

Answer:

A. 10, 15, 15, 18, 22, 25, 28, 35, 36

Step-by-step explanation:

Stem and leaf plot

1    |     0 5 5 8

2   |     2 5 8

3   |    5 6

Example : 1 | 0 means 10

So, considering the examples :

1    |     0 5 5 8

10,15,15,15,18

2   |     2 5 8

22,25,28

3   |    5 6

35,36

So, the data set is 10,15,15,15,18,22,25,28,35,36

So, Option A is true

Mind helping me out a bit? (Which of the rectangular boxes shown has the greater volume?) (Will give Brainliest if you are the first person, and IF YOUR CORRECT) ^.^

Answers

Answer:

Both are equal in volume

Select the correct solution in each column of the table.
Solve the following equation.
18
12 + 6s
Examine the given table, and select the accurate number of each type of solution, as well as any real solutions that exist.
Number of
Real Solutions
Number of
Extraneous
Solutions
Real Solutions
0
0
x = 0
1
1
x= 3
2
x = 0, x= 3
N

Answers

Answer:

Number of  Real Solutions = 1

Number of  Extraneous Solutions = 1

Real Solutions = (x = 3)

A smart-phone is thrown upwards from the top of a 240-foot building with an initial velocity of 32
feet per second. The height h of the smart-phone after t seconds is given by the quadratic equation
h = - 16t2 + 32t + 240. When will the smart-phone hit the ground?

Answers

Answer:

t = 5 sec

Step-by-step explanation:

h = - 16t² + 32t + 240

On the ground h = 0.

- 16t² + 32t + 240 = 0 / : (- 16)

t² - 2t - 15 = 0

(t-5)(t+3) = 0

t = 5, t = - 3

Time can be only positive, so t = 5 sec

Which equation is a function of x?
A. x = 5
B. x = y squared + 9
C. x squared = y
D. x squared = y squared + 16

Answers

Answer:

C: X^2= Y

I got it right on Edge

Answer:

Pretty sure its x2 =y

Step-by-step explanation:

solve this system of linear equations, separate the x- and y-values with a comma. -4x=-60-19y -7x=-48-19y

Answers

Answer:

The solution is at (-4,-4).

Step-by-step explanation:

(-4x=-60-19y)-1          Multiply by -1

4x = 60 + 19y

-7x = -48 - 19y         Add both equations together

-3x = 12                   Divide both sides by -3

x = -4

Plug this into the first equation to find the value of y

-4(-4) = -60 - 19y        Multiply

16 = -60 - 19y

+60  +60                    Add 60 to both sides

76 = -19y                    Divide both sides by -19

y = -4

pls help on this question
*image

Answers

Answer:

216 2/3

Step-by-step explanation:

yoys : frents

6   :   13

We have 100 yoys

100/6 =50/3

Multiply by 50/3

yoys      : frents

6 *50/3  :   13*50/3

100         : 650/3

As a mixed number

650/3 frents

216 2/3

The math club needs to raise more than $552.50 for a trip to the state competition. The club
has raised 12% of the funds. Which inequality shows how much money each of the 7 club
members needs to raise if each raises the same amount?
A
m< $69.45
В
m> $69.45
C
m<$72.35
D
m> $72.35

Answers

Answer:

A or B

Step-by-step explanation:

Leaning more towards B- u want the club to earn more not less

The correct inequality is m> $69.45

What is Inequality?

Mathematical expressions with inequalities are those in which the two sides are not equal. Contrary to equations, we compare two values in inequality. Less than (or less than or equal to), greater than (or greater than or equal to), or not equal to signs are used in place of the equal sign.

Given:

The math club needs to raise more than $552.50 for a trip to the state competition.

The club has raised 12% of the funds.

To represent the inequality we have to perform the following function

=552.50 x 0.12

=66.3

The amount remained= 552.50 - 66.3 = $486.2

So, for the 7 members = 486.2/7

                                     = $69.45

Hence, the inequality is m>$69.45

Learn more about inequality here:

https://brainly.com/question/28823603

#SPJ5

Can you help me out on this question?

Answers

Answer:

4. x= 18

Step-by-step explanation:

We solve the problem like this:

Since 3 is proportional to 6 and 9 is proportional to x, we set up a proportion.

3/9 = 6/x  

We then use cross-multiplication to solve for x.

The surface area of a cube is 24 square inches.
What is the side length of the cube?

Answers

Answer:

2 inches

Step-by-step explanation:

A cube has 6 faces. Therefore, each face is 4 square inches. (24/6) The side length of the square (face of the cube) is the square root of the area. The square root of 4 is 2. There is your answer :)

I need help with this guys

Answers

Answer:

25:53

Step-by-step explanation:

75:159 have a common factor of 3

75/3=25

159/3=53

One angle is
angle is 65 degrees, the other is
90 degrees, what is my missing angle?
What is my first and last name?

Answers

Answer:

your missing angle is

90+65 = 155

180-155 = 25

so your missing angle is 25 degrees

Other Questions
Venera sent a chain letter to her friends, asking them to forward the letter to more friends.The relationship between the elapsed time t, in months, since Venera sent the letter, and the number ofpeople, P(t), who receive the email is modeled by the following function:3t+7P(t) = 2Complete the following sentence about the monthly rate of change in the number of people who receivethe email.Round your answer to two decimal places.Every month, the number of people who receive the email is multiplied by a factor of Which equation is equivalent to(1/3)^x= 27^X+2 The length of a rectangle is 2 units greater than the width. The area of the rectangle is 24 square units. What is it's width? plz plz plz i really need this will mark biranleastTriangle ABC is to be reflected across line m to create triangle A'B'C'. Which best describes the location of B'?4 units to the right of line m.4 units to the left of line m.8 units to the right of line m.8 units to the left of line m. What is the solution to the system of equations?2x - y = 7y = 2x + 3 (2, 3) (2.7) no solution infinite number of solutions do rainforests have hot summers and cold winters? a) Find the square number712202527b) Find the cube number712202527 ATP contains a large amount of energy because _____. the two-way table below shows the intended careers among the incoming class of first-year college students, divided by gender. If a student is chosen and random, what is the probability of the student is female who intends to be a research scientist? Complete the first and second sentences, choosing the correct answer from the given ones.1. The water temperature in the dish depends on the A / B / C / D.A. average kinetic energy of water moleculesB. total kinetic energy of water moleculesC. water mass. D. potential energy of the container with water2. The internal energy of the water in the vessel is E / F / G.E. potential energy of the vessel with waterF. average kinetic energy of water moleculesG. sum of kinetic energy and potential water molecules HELP ME PLZ!!What is the maximum number of Go Math textbooks that can fit in Tier 1 given the measurements of the textbook below? Your answer must have work on how you did it OR an explanation. Explain how love is formed in life. How do you think the immigrant girl feels about being asked how to wash stairs? Check all that apply. A survey taken in a large statistics class contained the question: "What's the fastest you have driven a car (in miles per hour)?" The five-number summary for the 87 males surveyed is: min = 55, Q1 = 95, Median = 110, Q3 = 120, Max = 155 Should the largest observation in this data set be classified as an outlier? No Yes Why was Mount Olympus important to the Greeks? Solve the equation.y + 6 = 3y + 26 a certain Forest covers an area of 2000 square kilometers. Suppose that each year this area, decreases by 6%. What is the equation that best represents the area of the forest each year? A hot dog has about 1/4 the amount of protein as 3 ounces of hamburger. Together, they have about 25 grams of protein. How many grams of protein are in a 3 oz hamburger? The side length of a square patio is 2.59 meters. Which is the best estimate for the perimeter of the patio, in meters? Use the Intermediate Value Theorem to show that there is a root of the given equation in the specified interval.x4 + x ? 7 = 0, (1, 2)f(x) = x4 + x ? 7is (FILL IN)a) defined b) continuous c) negative d) positive on the closed interval [1, 2],f(1) = ?? FILL IN , and f(2) = ?? FILL INSince ?5 < FILL IN a)? b)? c)? d)0 < 11, there is a number c in (1, 2) such thatf(c) = FILL IN a)? b)? c)0 d)11 e)-5by the Intermediate Value Theorem. Thus, there is a FILL IN a) limit b)root c) discontinuity of the equationx4 + x ? 7 = 0in the interval (1, 2).